Coordination Compounds  MCQs

MCQs of Coordination Compounds

Showing 201 to 210 out of 226 Questions
201.
If six strong monodentate ligands combine with a metal ion with electronics configuration d6, what will be the hybridization ?
(a) sp3
(b) dsp2
(c) d2sp3
(d) sp3d2
Answer:

Option (c)

202.
If strong ligands have combined with a metal ion having electronic configuration d8, the magnetic property will be _____ .
(a) paramagnetic
(b) ferromagnetic
(c) ferrimagnetic
(d) diamagnetic
Answer:

Option (d)

203.
The correct statement about [FeF6]3- and [CoF6]3- ions is _____ .
(a) both have octahedral structure with equal magnetic moment
(b) both contain respectively Fe3+ and Co3+ metal ions and they have equal number of 3d-electrons
(c) sp3d2 hybridization exist in both and magnetic moment of [FeF6]3- is greater than that of [CoF3]3-
(d) both are coloured and have equal stability
Answer:

Option (c)

204.
The incorrect statement regarding d2sp3 hybridization is _____ .
(a) it involves six orbitals and each orbital has equal energy
(b) all these orbitals are on X-, Y- and Z-axes to one another
(c) they are directed towards eight corners of octahedral from the centre
(d) the angle between any two nearly orbitals is 90°
Answer:

Option (c)

205.
Which of the following 3d-electronic configuration always causes diamagnetic property ?
(a) d0 and d10
(b) d5 and d9
(c) d0 and d4
(d) d6 and d8
Answer:

Option (a)

206.
Select the correct formula for traiamminediaqua chlorido cobalt (III) chloride.
(a) [CoCl (NH3)3 (H2O)2] Cl2
(b) [Co (NH3)3 (H2O)Cl2] Cl
(c) [Co (NH3)3 (H2O)Cl3]
(d) [Co (NH3)3 (H2O)2]Cl3
Answer:

Option (a)

207.
For [FeIII(H2O)4 Cl2]Clx, x= _____ .
(a) 1
(b) 2
(c) 3
(d) 4
Answer:

Option (a)

208.
For [PtIVCl(NO2)2 (NH3)3]x, x= _____ .
(a) +1
(b) -1
(c) -2
(d) +2
Answer:

Option (a)

209.
For Mg(x) [CoII(CN)4(AcO)2]2, x= _____ .
(a) 1
(b) 2
(c) 3
(d) 4
Answer:

Option (d)

210.
In K3[FeIII(CN)x (C2O4)y], the values of x and y are respectively _____ . (I) x=4,y=1 (II) x=2, y=2 (III) x=2,y=4 (IV) x=4,y=4
(a) I and II
(b) I and IV
(c) I and III
(d) I,II and III
Answer:

Option (a)

Showing 201 to 210 out of 226 Questions